Questions tagged [analytic-number-theory]

A beautiful blending of real/complex analysis with number theory. The study involves distribution of prime numbers and other problems and helps giving asymptotic estimates to these.

Filter by
Sorted by
Tagged with
11 votes
1 answer
1k views

Conditional convergence of $\sum_{n\geq 1} \frac{\sin(p(n))}{n}$?

The series $\sum_{n\geq 1} \frac{\sin n}{n}$ is easily seen to be conditionally convergent, e.g. by Abel summation. But how about $\sum_{n\geq 1} \frac{\sin(n^2)}{n}$? (for which Abel summation fails)...
H. H. Rugh's user avatar
11 votes
2 answers
1k views

Number of representations of an integer as an (arbitrary) sum of products

If $n$ is a positive integer, let $r(n)$ denote the number of representations of $n$ as a sum of products of pairs of positive integers. (Here, the order of the terms in the sum does not matter, but ...
David Ellis's user avatar
11 votes
1 answer
1k views

Lower bound for exponential sums

Let $D$ be a subset of $\mathbb Z/n \mathbb Z$ containing $0$. For $m$ an integer, set $$\alpha(m,D)=\sum_{d \in D} e\left (\frac{m d }{n}\right ),$$ where as usual $e(x) = e^{2 i \pi x}$ This is an ...
Joël's user avatar
  • 25.7k
11 votes
1 answer
1k views

What might the (normalized) pair correlation function of prime numbers look like?

Cross-posting from Math.Stackexchange. You might have read about the fortuitous meeting between Montgomery and Dyson. The background is that the nontrivial zeros of the Riemann zeta function, when ...
anon's user avatar
  • 441
11 votes
2 answers
611 views

Methods to bound the number of solutions to $x^x \equiv 1 \mod p$ with $1 \le x \le p$

For a prime $p$, let $N(p)$ be the number of solutions $1 \le x \le p$ to $x^x \equiv 1 \mod p$. I am interested in methods to bound $N(p)$. Background: This quantity appears in Problem 1 of the ...
Christian Bernert's user avatar
11 votes
1 answer
594 views

Riemann zeta function: pair correlations vs. neighbor spacings

Montgomery's pair correlation conjecture states that the distribution of the pair correlations of the zeroes of the Riemann zeta function (normalized to have average spacing 1) is given by the ...
Kurisuto Asutora's user avatar
11 votes
3 answers
694 views

Counting points on lattices

I expect that the following is a standard problem from analytic number theory, but I don't know where exactly to look for an answer. Let f: ℤr→ H be a surjective homomorphism into a ...
Tzanko Matev's user avatar
11 votes
1 answer
307 views

Critical points of Dirichlet L functions

Let $L(s,\chi)$ denote a Dirichlet $L$-function for a real-valued non-principal character $\chi$. This has limiting value $L(\infty,\chi) = 1$ and we are interested in how this limit is approached ...
user2052's user avatar
  • 1,401
11 votes
2 answers
2k views

Good books on Dirichlet's class number formula

I refrained from asking the technical questions; maybe everyone didn't like my attitude. At least, help me finding good books. Can anyone suggest a good book that gives a complete reference to "...
user avatar
11 votes
1 answer
649 views

What is the motivation behind Ramanujan's conjecture?

One motivation I have seen given for Ramanujan's conjecture for the estimate $$ |a_p|< C p^{k - \frac{1}{2}} $$ for the Fourier coefficients of a cusp form of weight $2k$ is that it allows one to ...
Keivan Karai's user avatar
  • 6,104
11 votes
1 answer
1k views

The Bombieri Vinogradov Theorem restricted to moduli divisible by $k$

The Bombieri-Vinogradov Theorem states that given $A>0$, there exists $B>0$ such that for $Q=\sqrt{x}\left(\log x\right)^{-B},$ we have $$\sum_{q\leq Q}\max_{y\leq x}\max_{\begin{array}{c} a\...
Eric Naslund's user avatar
  • 11.3k
11 votes
1 answer
455 views

Siegel--Walfisz for number fields

For a number field $K$, we write $\Delta_K$ for its absolute discriminant. I was hoping for a Siegel--Walfisz type theorem of the following type: Let $A > 0$. Then for every $X > 0$, every ...
P. Koymans's user avatar
11 votes
1 answer
1k views

Smooth functions that resemble random walks

If the Riemann hypothesis holds, then the Mertens function $M(n)\equiv\sum_{x\leq n} \mu(n)$ behaves much like a 1D random walk. This includes the statements that $M(n)$ changes sign infinitely often ...
Curt von Keyserlingk's user avatar
11 votes
1 answer
259 views

Partial product of Euler factors

Let $\mathbb P$ denote the set of prime numbers and for a subset $T\subset \mathbb P$ let $$ \zeta_T(s)=\prod_{p\in T}\frac1{1-p^{-s}}, $$ where $\mathrm{Re}(s)>1$. Is there any $T$ such that $T$ ...
user avatar
11 votes
1 answer
1k views

Prime Power Gaps

In 2000, Baker, Harman and Pintz proved that there is always a prime in the interval $(n-n^{0.525}, n)$. There are also conditional results implying smaller intervals. Nevertheless, I could not find ...
Ami Paz's user avatar
  • 375
11 votes
1 answer
436 views

Are some numbers more equidistributed than others?

Weyl's theorem says that $n\alpha$ is equidistributed mod 1 for any irrational $\alpha$. One corollary is that, if I consider the fractional part $\{n\alpha\}$ for $n \leq N$, and look at the indices ...
Elena Yudovina's user avatar
11 votes
1 answer
481 views

Second moment estimates for $\zeta(s)$: different methods?

What are some different ways to achieve the bound $$\int_0^T \left|\zeta\left(\frac{1}{2} + i t\right)\right|^2 dt = T \log \frac{T}{2 \pi} + (2 \gamma - 1) T + E(T)$$ with an error term $E(T) = O(T^{\...
H A Helfgott's user avatar
  • 19.4k
11 votes
1 answer
661 views

Poisson summation formula for number fields

Poisson summation formula is widely used in many parts of the litterature, its classical formulation for sums over integers as well as its adelic version. What is its corresponding form for more ...
Automorphic's user avatar
11 votes
0 answers
236 views

Sums of $\Lambda(n)$ or $\mu(n)$ with hyperbolic-function weights: surprise!

I was leafing through Gradshteyn–Ryzhik in bed yesterday, as one does, and noticed on the last page that the Mellin transforms of several hyperbolic functions have a factor of $\zeta(s-1)$ or $\zeta(s)...
H A Helfgott's user avatar
  • 19.4k
11 votes
0 answers
424 views

Growth of $n=n(k)$ for which there's a non-trivial solution to $x_1^k+\cdots+x_n^k=y^k$?

Walter Hayman just asked me the following question. What, if anything, is known about the growth of the function $n(k)$, where $k\geq1$ is an integer, and $n=n(k)\geq2$ is the smallest integer for ...
Kevin Buzzard's user avatar
10 votes
3 answers
3k views

A number encoding all primes

This may be a soft question, but it's just something I thought of one night before sleeping. It's not my field at all, so I am just asking out of curiosity. Has anyone studied the number which is the ...
David Carchedi's user avatar
10 votes
2 answers
903 views

$\psi(x)-x$ on average

This is a reference question: Let $\psi(x)$ be the psi-Chebyshev function. Is there any unconditional result in the literature that proves that there exists $0<a<2$ such that $$ \int_2^x (\psi(y)...
Dr. Pi's user avatar
  • 2,949
10 votes
4 answers
1k views

An interesting sum over lattice points in a large disk centered at the origin

Evaluate the the limit, as $r \rightarrow \infty $, of the sum $\displaystyle \sum \limits_{(m,n) \in D_r}$ $\displaystyle (-1)^{m+n} \over \displaystyle m^2 + n^2$, where $D_r$ denotes the closed ...
Wahome's user avatar
  • 737
10 votes
2 answers
943 views

Modular forms with finitely many or very few non-zero Fourier coefficients

I have an elementary question on modular forms, but which I don't know how to solve. a) Is there a congruence subgroup $\Gamma \leq \mathrm{SL}_2(\Bbb Z)$, an integer $k \in \Bbb Z$ and a non-...
Alphonse's user avatar
  • 255
10 votes
1 answer
971 views

A generalisation of theorem of Landau on sum of two squares?

Let $r(B)$ be the number of integers $1 \leq n \leq B$ such that $n = x^2 + y^2$ for some $x, y \in \mathbb{Z}.$ Then it is a known theorem of Landau that $$ r(B) \sim C \frac{B}{\sqrt{\log B}} $$ ...
Johnny T.'s user avatar
  • 3,595
10 votes
3 answers
1k views

Explicit formula for elementary symmetric sum

For $k\ge1$, $j\ge1$, Let $$e_k(j)=\sum_{1\le i_1<...<i_k\le j}i_1\cdot\cdot\cdot i_k.$$ We know that $e_k(j)$ is a polynomial in $j$ with coefficients depending on $k$. I am curious about ...
mygreatwall's user avatar
10 votes
3 answers
2k views

Natural density of set of numbers not divisible by any prime in an infinite subset

Suppose $S$ is a subset of the primes with natural density $0 < \alpha < 1$ within the primes. If $$D(X) := \{n \leq X \mid p \not \mid n \text{ for all } p \in S \}$$ (so $D(X)$ is numbers at ...
bean's user avatar
  • 479
10 votes
2 answers
846 views

Bound on gcd of two integers

Well this is a problem I was fiddling with. I came up with it but it probably is not original. Suppose $a\in \mathbb{N}$ is not a perfect square. Then show that : $$\text{gcd}(n,\lfloor{n\sqrt{a}}...
shadow10's user avatar
  • 1,090
10 votes
3 answers
928 views

What is the intuition behind applying the Mellin Transform to prime distribution?

I am an undergraduate student writing an expository thesis on the complex-analytic proof of the Prime Number Theorem. I understand that applying the Mellin Transform to the partial sum of the van ...
onionbread's user avatar
10 votes
7 answers
850 views

$\int_L^\infty \exp(- t - y/t) \, dt = \text{?}$

Let $y>0$, $L>0$. Has the (special?) function given by $$f(y,L) = \int_{L}^\infty e^{- t - y/t} \, dt$$ been studied? Are there precise, simple bounds? Let me try to attempt to reinvent the ...
H A Helfgott's user avatar
  • 19.4k
10 votes
2 answers
1k views

Are the ideles literally a Picard group?

I understand that in the number field / function field analogy, the ideles $\mathbb I_K$ of a number field $K$ are supposed to be analogous to the Picard group of a function field. Question: Is this ...
Tim Campion's user avatar
  • 61.6k
10 votes
2 answers
1k views

Sums of two squares: What is known about the distribution of r(n)?

The distribution of sums of two squares has been studied by Landau. What is known about the distribution of the function $r(n)$, the number of representations of $n$ as the sum of two squares? Some ...
Yuri Gurevich's user avatar
10 votes
1 answer
456 views

Sign of the function $f(n)=\sum_{k=1}^n\frac{\mu(k)}{k}$

It is well-known that the Mertens function $M(n)=\sum_{k=1}^n\mu(k)$ changes sign infinitely many times when $n\rightarrow +\infty$. Let $f(n)=\sum_{k=1}^n\frac{\mu(k)}{k}$, then $\lim\limits_{n\...
ZZP's user avatar
  • 424
10 votes
5 answers
2k views

Riemann–Von Mangoldt formula

Let $$N(T) = \#\{\rho \in \mathbb{C}: \zeta(\rho) = 0,\, \operatorname{Im} \rho \in (0,T]\}$$ denote the number of zeros of $\zeta(s)$, counting multiplicities, with imaginary part lying in the ...
Jesse Elliott's user avatar
10 votes
1 answer
628 views

Regularized sums of Mobius sequence

Do $\lim_{s \rightarrow \infty} \sum_{n \geq 1} \mu(n) e^{-n/s}$ and $\lim_{s \rightarrow \infty} \sum_{n \geq 1} \mu(n) e^{-n^2/s^2}$ both equal $-2$? Experimentally this seems plausible (up through ...
James Propp's user avatar
  • 19.4k
10 votes
3 answers
2k views

What is the relationship between the Bell numbers, the Bell polynomials, and the partition numbers?

A friend of mine and I were wondering what relationship exists between the partition numbers $p_{n}$ and the Bell numbers $B_{n}$ (and also possibly the Bell polynomials $B_{n,k}(x_1,x_2,\dots,x_{n-k+...
graveolensa's user avatar
10 votes
1 answer
664 views

Set of prime numbers $q$ such that $\sum\limits_{p\leq\sqrt{q}}p=\pi(q)$, where $p$ are prime numbers

The question is: does the set of prime numbers $q$ such that $\sum\limits_{p\leq\sqrt{q}}p=\pi(q)$, where $p$ are prime numbers, contain infinitely many elements? You can find the first elements here (...
Juan Moreno's user avatar
10 votes
1 answer
708 views

Why are the coefficients of the modular equation so large?

The modular equation $\Phi_n(X,Y)$ is a polynomial in $\mathbf Z[X,Y]$ relating the modular invariant $j$ and the functions $j\left(\frac{a\tau+b}{c\tau+d}\right)$, where $ad-bc=n$. For example, we ...
Shimrod's user avatar
  • 2,335
10 votes
3 answers
1k views

Quantitative and elementary proofs of the Prime Number Theorem

I would like to know two things: one, whether the best quantative bounds in the Prime Number Theorem are still basically those given by the Vinogradov-Korobov zero-free region? and two, whether there ...
user36212's user avatar
  • 1,687
10 votes
2 answers
678 views

Averages over integer points of the sphere

A paper of William Duke proves that integer points on the sphere are equidistributed: $$ V_n = \{ (x,y,z) \in \mathbb{Z}^2 : x^2 + y^2 + z^2 = n \}. $$ Up to reflections across the $x$, $y$ and $z$ ...
john mangual's user avatar
  • 22.6k
10 votes
1 answer
2k views

Is new $n$-conjecture as follows correct?

Given a positive integer $P>1$, let its prime factorization be written as$$P=p_1^{a_1}p_2^{a_2}p_3^{a_3}\cdots p_k^{a_k}.$$ Define the functions $h(P)$ by $h(1)=1$ and $h(P)=\min(a_1, a_2,\ldots,...
Đào Thanh Oai's user avatar
10 votes
1 answer
814 views

Infinitely many primes that split completely in an arithmetic progression

Let $d \geq 1$ be an integer. Dirichlet's theorem on arithmetic progression implies that the arithmetic progression $a, a+d, a+2d, \ldots$ contains infinitely many primes if and only if $\gcd(a,d)=1$. ...
Xiao Xiao's user avatar
  • 103
10 votes
1 answer
1k views

How does Riemann hypothesis implies estimates?

In Iwaniec, Luo and Sarnak article (precisely (4.23)), it is said that GRH for $L(s, \mathrm{sym}^2(f))$, for a holomorphic cusp newform $f$ of level $N$ and weight $k$, implies $$\sum_{p \nmid N} \...
Wolker's user avatar
  • 541
10 votes
1 answer
296 views

Coefficient bounds on cusp forms, half-integer weight

Let $f(\tau) = \sum_{n=1}^{\infty} a(n) q^n$ be a cusp form on $\Gamma_0(4N)$ of half-integer weight $k \ge 5/2.$ The Ramanujan-Petersson conjecture in this case is that $$a(n) \ll n^{(k-1)/2 + \...
user105068's user avatar
10 votes
1 answer
500 views

An upper bound for the length of the continued fraction expansion of $\sqrt d$

Let $d\ge 2$ and let $$ \sqrt d =[a_0; \overline{a_1,\dots, a_\ell, 2a_0}] $$ be its continued fraction expansion. Clearly, if $d=n^2+1$, then $\ell=0$, which gives the lower bound for $\ell$. ...
Nikita Sidorov's user avatar
10 votes
2 answers
3k views

least prime in a arithmetic progression

Hello Here I want to consider the simplest arithmetic progression $n\equiv 1\pmod{q}$ where $q$ is a prime. Is it true that we can find a prime $p\leq q^2$ in this arithmetic progression? This ...
M.B's user avatar
  • 2,468
10 votes
1 answer
472 views

Asymptotic behavior of a "strange" arithmetic function

Write $f(n)$ for the quotient of $n$ by its largest squarefree divisor. In other words, $f$ is a multiplicative function with $f(p^k) = p^{k-1}$ for all $k \geq 1$. What, if anything, is known about ...
JSE's user avatar
  • 19.1k
10 votes
1 answer
532 views

Is $j(\tau)^{1/3}$ the hauptmodul for the congruence subgroup generated by $\tau\rightarrow\tau+3, \tau\rightarrow-1/\tau$?

The 3rd root of the modular invariant $j$ is $$ j(\tau)^{1/3}=q^{-1/3}(1+ 248q+ 4124q^2+ 34752q^3+\cdots),$$ where $q=e^{2\pi i \tau}$. I was wondering if $j(\tau)^{1/3}$ the hauptmodul for the ...
Franklin Wu's user avatar
10 votes
1 answer
594 views

Statement of the pair correlation conjecture

In his paper "The pair correlation of zeros and the zeta function", Montgomery defines a function $$F(\alpha,T) = \left(\frac{T}{2 \pi} \log T\right)^{-1} \sum_{0 < \gamma, \gamma' < T} T^{i \...
Joël's user avatar
  • 25.7k
10 votes
1 answer
458 views

A basic estimate of exponential sums

Demeter in his book "Fourier Restriction, Decoupling, and Applications" (P287) used the following estimate: \begin{equation} \sup_{0\leq n\leq q}\bigg|\sum_{m=0}^n e^{2\pi i\frac{a}{q}m^2}\...
Dapao Zhang's user avatar

1
7 8
9
10 11
59